Đến nội dung

shinichikudo201 nội dung

Có 473 mục bởi shinichikudo201 (Tìm giới hạn từ 09-06-2020)



Sắp theo                Sắp xếp  

#449604 $\frac{a}{(ab+a+1)^{2}}+\frac...

Đã gửi bởi shinichikudo201 on 12-09-2013 - 15:21 trong Bất đẳng thức và cực trị

Mình có bài này mong các bạn giải giùm:

Cho a, b, c là các số dương thoả mãn abc=1. Chứng minh:

$\frac{a}{(ab+a+1)^{2}}+\frac{b}{(bc+b+1)^{2}}+\frac{c}{(ca+c+1)^{2}}\geq \frac{1}{a+b+c}$

Mình đang học lớp 8.

Thanks




#449606 Chứng minh $ab(a+b)^{2}\geq \frac{1}{...

Đã gửi bởi shinichikudo201 on 12-09-2013 - 15:26 trong Bất đẳng thức và cực trị

Mình có bài này mong các bạn giải giùm:

Cho a,b thỏa mãn $\sqrt{a}+\sqrt{b}=1$. Chứng minh:

$ab(a+b)^{2} \leq \frac{1}{64}$

Mình đang học lớp 8, các bạn dùng kiến thức lớp 8 nhé.

Thanks.




#449607 Chứng minh $\frac{1}{a^{2}+2b^{2...

Đã gửi bởi shinichikudo201 on 12-09-2013 - 15:31 trong Bất đẳng thức và cực trị

Mình có bài này mong các bạn giải giùm:

Chứng minh rằng với mọi số thực dương a, b, c thỏa mãn abc=1:

$\frac{1}{a^{2}+2b^{2}+3}+\frac{1}{b^{2}+2c^{2}+3}+\frac{1}{c^{2}+2a^{2}+3}\leq \frac{1}{2}$

Mình đang học lớp 8 nhé.

Thanks

 




#449609 Chứng minh $\frac{1}{x^{3}+y^{3}...

Đã gửi bởi shinichikudo201 on 12-09-2013 - 15:38 trong Đại số

Mình có bài này mong các bạn giải giùm:

Cho x, y là các số thực dương thỏa mãn x+y=1. Chứng minh

$\frac{1}{x^{3}+y^{3}}+\frac{1}{xy}\geq 4+2 \sqrt{3}$

Mình đang học lớp 8 nhé.

Thanks




#449610 Tìm GTLN của biểu thức: $P=\frac{z^{4}}{1+...

Đã gửi bởi shinichikudo201 on 12-09-2013 - 15:46 trong Bất đẳng thức và cực trị

Mình có bài này mong các bạn giải giùm:

Cho x, y, z dương thỏa mãn: $xy^{2}z^{2}+x^{2}z+y=3z^{2}$. Tìm Max của biểu thức

$P=\frac{z^{4}}{1+z^{4}(x^{4}+y^{4})}$

Mình đang học lớp 8 nhé.

Thanks




#449611 Tìm Max của biểu thức $f(x)=\sqrt{2x^{2}+5x+2}+...

Đã gửi bởi shinichikudo201 on 12-09-2013 - 15:51 trong Đại số

Mình có bài này mong các bạn giải giùm:

Cho x$\geq- \frac{1}{2}$. Tìm max của biểu thức:

$f(x)=\sqrt{2x^{2}+5x+2}+2\sqrt{x+3}-2x$

Mình đang học lớp 8 nhé.

Thanks




#449612 Tìm các cặp số dương (x;y) sao cho xy đạt giá trị nhỏ nhất

Đã gửi bởi shinichikudo201 on 12-09-2013 - 15:56 trong Đại số

Mình có bài này mong các bạn giải giùm:

Tìm các cặp số dương (x;y) thỏa mãn $2x^{2}+2y^{2}-x^{2}y^{2}-6xy-4x+4y+10=0$ để tích xy đạt giá trị nhỏ nhất.

Mình đang học lớp 8 nhé.

Thanks




#449614 Tìm Min của biểu thức: $f(x)=(x-1)(x+6\sqrt{x}+8) (x...

Đã gửi bởi shinichikudo201 on 12-09-2013 - 16:01 trong Đại số

Mình có bài này mong các bạn giải giùm:

Tìm giá trị nhỏ nhất của biểu thức: $f(x)=(x-1)(x+6\sqrt{x}+8) (x\geq 0)$

Mình đang học lớp 8 nhé.

Thanks




#449615 Cho a>b>0. So sánh 2 số x; y với $x=\frac{1+a}...

Đã gửi bởi shinichikudo201 on 12-09-2013 - 16:05 trong Đại số

Mình có bài này mong các bạn giải giùm:

Cho a>b>0. So sánh 2 số x; y với

$x=\frac{1+a}{1+a+a^{2}}$ và $y=\frac{1+b}{1+b+b^{2}}$

Mình đang học lớp 8 nhé.

Thanks




#449642 Tìm các cặp số dương (x;y) sao cho xy đạt giá trị nhỏ nhất

Đã gửi bởi shinichikudo201 on 12-09-2013 - 17:47 trong Đại số

Ta có $x^{2}y^{2}-6xy=2.(x-1)^2+2.(y-1)^2+6 \geq 6 nên x^{2}y^{2}-6xy-6\geq 0$

Rồi sao hả bạn???




#449646 $\frac{a}{(ab+a+1)^{2}}+\frac...

Đã gửi bởi shinichikudo201 on 12-09-2013 - 17:55 trong Bất đẳng thức và cực trị

Lời giải. Ta có $ab+a+1=ab+a+abc=a(bc+b+1)$. Do đó $\frac{a}{(ab+a+1)^2}= \frac{1}{a(bc+b+1)^2}$. Tương tự và ta thu được $\sum \frac{a}{(ab+a+1)^2}= \sum \frac{1}{a(bc+b+1)^2}$. Áp dụng BĐT Cauchy-Schwarz ta có $$\left( \sum \frac{1}{a(bc+b+1)^2} \right) \cdot (a+b+c) \ge \left( \sum \frac{1}{bc+b+1} \right)^2=1$$

Vậy $\sum \frac{a}{(ab+a+1)^2} \ge \frac{1}{a+b+c}$.

Dấu đẳng thức xảy ra khi và chỉ khi $a=b=c=1$.

Bạn ơi. Mình đang học lớp 8 nhé.




#449656 Định nghĩa Sigma đại số $\sum$

Đã gửi bởi shinichikudo201 on 12-09-2013 - 18:21 trong Đại số

Các bạn làm ơn cho minh hỏi về định nghĩa $\sum$ với (mình học lớp 8)

Thanks

 




#449713 $\frac{a}{(ab+a+1)^{2}}+\frac...

Đã gửi bởi shinichikudo201 on 12-09-2013 - 21:35 trong Bất đẳng thức và cực trị

Cơ bản là bạn đâu có hỏi bài lớp 8  :mellow:  Chỗ nào bạn không hiểu cứ hỏi.

Tại sao có $(\sum \frac{1}{bc+b+1})^{2}=1$ hả bạn?




#449734 $\frac{a}{(ab+a+1)^{2}}+\frac...

Đã gửi bởi shinichikudo201 on 12-09-2013 - 22:05 trong Bất đẳng thức và cực trị

:luoi: chắc là không hiểu cái dòng thứ $3$ mà vế phải là $1$ đó , bạn giải thích nốt đi (p/s : lớp $8$ mà chơi bài này là không vừa )

Bạn giảng hộ mình chỗ này nhé.

cơ bản nếu lớp 8 thì sẽ không có bài này
$\sum$ là tổng có 2 dạng, cái này hỏi bangbang1412

Mình hỏi rồi bạn nhé. Mình chỉ chưa hiểu dòng 3 thôi.



#449740 Chứng minh $ab(a+b)^{2}\geq \frac{1}{...

Đã gửi bởi shinichikudo201 on 12-09-2013 - 22:12 trong Bất đẳng thức và cực trị

....................................................................................................................................

                    

:angry: Nhưng bài này không đúng rồi cho $a=0,b=1$ thì sai 

Sao lại vậy bạn? a=0 thì tích = 0 rồi còn gì? 




#449746 Định nghĩa Sigma đại số $\sum$

Đã gửi bởi shinichikudo201 on 12-09-2013 - 22:17 trong Đại số

Ký hiệu này có hai loại là $\sum_{cyc}^{}$ và $\sum_{sym}^{}$ , loại $cyc$ là tổng hoán vị , tổng $sym$ là tổng đối xứng .

Ví dụ $\sum_{cyc}^{cyc}a^{2}b=a^{2}b+b^{2}c+c^{2}a$

Còn $\sum_{sym}^{sym}a^{2}b=a^{2}b+b^{2}a+b^{2}c+c^{2}b+c^{2}a+a^{2}c$ 

Mình thấy trên này mọi người dùng không theo kí hiệu nào cả




#449750 Chứng minh $ab(a+b)^{2}\geq \frac{1}{...

Đã gửi bởi shinichikudo201 on 12-09-2013 - 22:18 trong Bất đẳng thức và cực trị

Ựa , thế nó mới sai , 

Xin lỗi mình nhầm đề. Đúng ra phải đổi chiều BĐT mới phải.




#449761 Chứng minh $ab(a+b)^{2}\geq \frac{1}{...

Đã gửi bởi shinichikudo201 on 12-09-2013 - 22:32 trong Bất đẳng thức và cực trị

Để cho dễ nhìn ta đặt $a=x^{2},b=y^{2}$ với $x,y\geq 0$ , ta có giả thiết $x+y=1$ và cần chứng minh

                                         $(xy)^{2}(x^{2}+y^{2})^{2}\geq \frac{1}{64}$

Lấy căn bậc hai của hai vế và ta chứng minh :

                                         $xy(x^{2}+y^{2})\geq \frac{1}{8}$

:angry: Nhưng bài này không đúng rồi cho $a=0,b=1$ thì sai 

Chứng minh tiếp như thế nào hả bạn???




#449768 Tìm GTLN của biểu thức: $P=\frac{z^{4}}{1+...

Đã gửi bởi shinichikudo201 on 12-09-2013 - 22:42 trong Bất đẳng thức và cực trị

Chia cả 2 vế của biểu thức đề bài cho z$^{2}$ rồi đặt $\frac{1}{z}$=t.Ta đưa về xy$^{2}$+x$^{2}$z+yz$^{2}$=3.Và P=$\frac{1}{x^4+y^4+z^4}$.Áp dụng bđt cosi cho 4 số ta có :$x^4+y^4+y^4+1 \geq 4xy^2,x^4+x^4+z^4+1\geq 4x^2z,z^4+z^4+x^4+1\geq 4z^2x.$Cộng theo vế suy ra $x^4+y^4+z^4 \geq 3$ nên P$\leq$$\frac{1}{3}$.Dấu = xảy ra khi x=y=z=1

Bạn làm mình chẳng hiểu gì cả !!!! Bạn lẫn lộn $t$ với $z$ hết rồi




#449779 Chứng minh $\sqrt{\frac{a+b}{c}}...

Đã gửi bởi shinichikudo201 on 12-09-2013 - 22:58 trong Bất đẳng thức và cực trị

Mình có bài này mong các bạn giải giùm:

Chứng minh với mọi a; b; c > 0 thì

$\sqrt{\frac{a+b}{c}}+\sqrt{\frac{b+c}{a}}+\sqrt{\frac{c+a}{b}}\geq 2(\sqrt{\frac{c}{a+b}}+\sqrt{\frac{a}{b+c}}+\sqrt{\frac{b}{a+c}})$

Mình đang học lớp 8 nhé. 

Thanks




#449784 Chứng minh rằng $\frac{x^{4}+1}{x^{3...

Đã gửi bởi shinichikudo201 on 12-09-2013 - 23:03 trong Bất đẳng thức và cực trị

Mình có bài này mong các bạn giải giùm:

Cho $x\epsilon \mathbb{R}$ thỏa mãn x>1. Chứng minh:

$\frac{x^{4}+1}{x^{3}-x}\geq 2\sqrt{2}$

Mình đang học lớp 8.

Thanks




#449787 Chứng minh rằng $\frac{a}{b}+\frac{b...

Đã gửi bởi shinichikudo201 on 12-09-2013 - 23:08 trong Bất đẳng thức và cực trị

Mình có bài này mong các bạn giải giùm:

Cho 3 số dương a;b;c. Chứng minh rằng:

$\frac{a}{b}+\frac{b}{c}+\frac{c}{a}\geq \frac{a+b+c}{\sqrt[3]{abc}}$

Mình đang học lớp nhé.

Thanks




#449791 Chứng minh rằng $\frac{x^{3}}{y^{3...

Đã gửi bởi shinichikudo201 on 12-09-2013 - 23:14 trong Bất đẳng thức và cực trị

Mình có bài này mong các bạn giải giùm:

Cho x; y; z>0. Chứng minh rằng:

$\frac{x^{3}}{y^{3}}+\frac{y^{3}}{z^{3}}+\frac{z^{3}}{x^{3}}\geq \frac{x^{2}}{y^{2}}+\frac{y^{2}}{z^{2}}+\frac{z^{2}}{x^{2}}$

Mình đang học lớp 8.

Thanks.

 




#449795 Chứng minh $\frac{1}{a^{3}+b^{3}...

Đã gửi bởi shinichikudo201 on 12-09-2013 - 23:20 trong Bất đẳng thức và cực trị

Mình có bài này mong các bạn giải giùm:

Cho a; b; c dương. Chứng minh:

$\frac{1}{a^{3}+b^{3}+abc}+\frac{1}{b^{3}+c^{3}+abc}+\frac{1}{a^{3}+c^{3}+abc}\leq \frac{1}{abc}$

Mình đang học lớp 8.

Thanks.




#449798 Chứng minh $\frac{ab}{a^{5}+b^{5...

Đã gửi bởi shinichikudo201 on 12-09-2013 - 23:27 trong Bất đẳng thức và cực trị

Mình có bài này mong các bạn giải giùm:

Cho a;b;c dương và abc=1. Chứng minh rằng:

$\frac{ab}{a^{5}+b^{5}+ab}+\frac{bc}{b^{5}+c^{5}+bc}+\frac{ca}{c^{5}+a^{5}+ac}\leq 1$

Mình đang học lớp 8 nhé.

Thanks.